Find the distance between the points (2, 4) and (8, -8) on a coordinate plane, to the nearest whole number.

A.7
B.11
C.13
D.16

Answers

Answer 1

Answer:

C

Step-by-step explanation:

distance between two points formula:

[tex]d = \sqrt{( x2 - x1) ^{2} + (y2 - y1) ^{2} } [/tex]

where the x and y values are derived from the known points

we are given the points (2, 4) and (8, -8)

given these two points let's define each variable

x1 = 2

x2 = 8

y1 = 4

y2 = -8

we now substitute in these values into the formula

[tex]d = \sqrt{(8 - 2) ^{2} + ( - 8 - 4)^{2} } [/tex]

now we evaluate the expression using PEMDAS

first we do the subtraction inside of the parenthesis

8 - 2 = 6

-8 - 4 = -12

d = sqrt(6)^2 + (-12)^2

next we do the exponents

6^2 = 36

-12^2 = 144

d = sqrt( 144 + 36 )

next do the addition inside of the parenthesis

144 + 36 = 180

d = sqrt ( 180 )

finally we do the square root of 180

sqrt ( 180 ) = 13 ( rounded to the nearest whole )

d = 13


Related Questions

Pls answer all questions

Answers

5:-c

The 3multiples are 25,50,75

6:-c

[tex]\\ \sf\longmapsto 6\:of\:2+9-5[/tex]

[tex]\\ \sf\longmapsto 6(11-5)[/tex]

[tex]\\ \sf\longmapsto 6(6)[/tex]

[tex]\\ \sf\longmapsto 36[/tex]

7:-a

12,17

8:-c

22=2×1188=2×2×2×11

HCF=2×2×11=44

9:-c

LCM of 9 and 5 is 45 hence its true

10:-c

HCF of two consecutive numbers is 1

I can’t figure out which formula goes with which sequence. Help……. Please???

1st sequence:
1,3,9,27,….
What is the formula?

2nd sequence:
400,200,100,50,…
What is the formula?

3rd sequence:
4,8,16,32,…
What is the formula?

4th sequence:
400,100,25,6.25,…
What is the formula?

5th sequence:
1,5,25,125,…
What is the formula?

6th sequence:
1000,500,250,125,…
What is the formula?

7th sequence:
2,10,50,250
What is the formula?

I need to know the correct geometric sequence formula for each of these sequences. Thanks!!!

Answers

For a geometric sequence

a, ar, ar ², ar ³, …

the n-th term in the sequence is ar ⁻ ¹.

The first sequence is

1, 3, 9, 27, …

so it's clear that a = 1 and r = 3, and so the n-th term is 3 ⁻ ¹.

The second sequence is

400, 200, 100, 50, …

so of course a = 400, and you can easily solve for r :

200 = 400r   ==>   r = 200/400 = 1/2

Then the n-th term is 400 (1/2) ⁻ ¹.

Similarly, the other sequences are given by

3rd: … 4 × 2 ⁻ ¹

4th: … 400 (1/4) ⁻ ¹

5th: … 5 ⁻ ¹

6th: … 1000 (1/2) ⁻ ¹

7th: … 2 × 5 ⁻ ¹

for all of the questions
with method
thankyou​

Answers

Answer:

(i) 7/10

(ii) 3/10

(iii) 1/5

(iv) Rs 40,000

Step-by-step explanation:

The fraction of the salary spent on food = 1/2

The fraction of the salary spent on rented house fee = 1/5

(i) The fraction spent for both food and rental fee = (1/2) + (1/5) = (5 + 2)/10 = 7/10

(ii) The remainder (rest) of the salary = 1 - 7/10 = 3/10

The fraction of the remainder spent for children's education = 1/3

The fraction of the total salary spent for the children's education = (1/3) × (3/10) = 1/10

(iii) The remaining portion deposited in the bank = 1 - (1/10 + 7/10)) = 2/10 = 1/5

(iv) The amount equal to portion of 1/5 of his salary deposited in the bank is Rs 8000

Let x represent his whole salary, we have;

(1/5) × x = Rs 8,000

x = 5 × Rs 8,000 = Rs 40,000

His whole salary is Rs 40,000.

Find the distance between the two points.
(-5,1)
(0,0)
[?]
Enter the number that
goes beneath the
radical symbol.

Answers

Answer:

[tex]{26}[/tex]

Step-by-step explanation:

[tex] \sqrt{(0 - ( - 5) ){}^{2} + (0 - 1) {}^{2} } [/tex]

[tex] = \sqrt{ {(5)}^{2} + {( - 1)}^{2} } [/tex]

[tex] = \sqrt{25 + 1} [/tex]

[tex] = \sqrt{26} [/tex]

Answered by GAUTHMATH

Answer:

26

Step-by-step explanation:

(-5, 1) (0, 0)

sqrt(5^2 + (-1^2))

sqrt(25 + 1)

sqrt(26)

So, 26 goes under the radical.

i have 17 coins. N of them are nickels and the rest are dimes. write an expression in two different ways for the amount of money that i have

Answers

Answer:

Step-by-step explanation:

(N)0.05 + (17-N)0.1 = M; M = amount of money I have.

Or 1.7-0.05N = M.

An expression two different ways to  the amount of money is equals to

1. 5N + 10 (17 -N)  = Y cents

2. N + 2(17 - N) = Y nickels

What is amount?

" Amount is defined as the total of any given quantity."

According to the question,

Total number of coins = 17

Number of nickels coins = N

Number of dimes coin = 17 - N

'Y' express the amount of money

Represent amount of money in cents

1 dime = 10 cents

1 nickel = 5 cents

Expression to represents amount of money in cents,

5N + 10 (17 -N)  = Y cents

Expression to represents amount of money in nickels,

1 dime = 2 nickel

N + 2(17 - N) = Y nickels

Hence, an expression two different ways to  the amount of money is equals to

1. 5N + 10 (17 -N)  = Y cents

2. N + 2(17 - N) = Y nickels

Learn more about amount here

https://brainly.com/question/27337635

#SPJ3

a trader is given 15% discount on goods bought from a factory. If the original price of an item in the factory is 45000. Calculate the amount the trader paid for them​

Answers

Answer:

38,250

Step-by-step explanation:

Its a 15% discount so youre only paying 85% of the original 45,000. You then multiply 45,000 by .85 to find what youre paying because the decimal version of 85% is .85. You can find the decimal version of a percentage by dividing the percentage by 100; 85/100=.85. Once you multiply 45,000 by .85 you get 38,250.

Find the length of the side labeled x. Round intermediate values to the nearest tenth. Use the rounded values to calculate the next value. Round your final answer to the nearest tenth.

A. 11.7
B. 11.9
C. 11.4
D. 14.4

Answers

Answer:

Option A, 11.7

Step-by-step explanation:

To find height,

tan 68 = 26/h

h = 26/tan(68)

to find x

tan(42) = h/x

x = h/tan(42)

x = 11.7 (rounded to the nearest tenth)

Answered by GAUTHMATH

11) MZHGF = 16x + 4, m EGF = 110°,
and MZHGE = 3x + 11. Find x.
G
H
F
E

Answers

Answer:  x = 9

=======================================================

Explanation:

The smaller angles HGE and EGF combine to form the largest angle HGF.

This is an example of the angle addition postulate.

(angle HGE) + (angle EGF) = angle HGF

(3x+11) + (110) = 16x+4

3x+121 = 16x+4

121-4 = 16x-3x

117 = 13x

13x = 117

x = 117/13

x = 9 which is the final answer

We can stop here.

--------------------

Extra info (optional section)

Use this x value to find each angle shown below

angle HGE = 3x+11 = 3*9+11 = 27+11 = 38 degreesangle HGF = 16x+4 = 16*9+4 = 144+4 = 148 degrees

Then notice how,

(angle HGE)+(angle EGF) = 38+110 = 148

which is exactly the measure of angle HGF

This confirms that the equation

(angle HGE) + (angle EGF) = angle HGF

is true for that x value of x = 9. Therefore, the answer is confirmed.

9514 1404 393

Answer:

  11) x = 9°

  13) x = -12°

Step-by-step explanation:

11) ∠HGE +∠EGF = ∠HGF

  (3x +11) +(110°) = (16x +4)

  117° = 13x . . . . . . . . . . . . . . . subtract 3x+4, collect terms

  9° = x . . . . . . . . . . . . divide by 13

__

13) ∠BCF +∠FCD = ∠BCD

  (x +78) +(x +41) = 95°

  2x = -24°

  x = -12°



I need help ASAP please

Answers

Answer:

it's the third one.

Step-by-step explanation:

It should be a bold/filled circle on the line instead of an empty one.

can anyone help me please ?

Answers

3 Answers: Choice B, C, and D

Basically, everything except choice A.

=========================================================

Explanation:

All exponential functions can be written into the form y = a*b^x

The b term determines if we have growth or decay.

If 0 < b < 1, then we have decay. If b > 1, then we have growth.

------------

For choice A, we have b = 1.7 which satisfies b > 1. This represents growth. So we cross choice A off the list.

Choice B looks almost identical since it appears b = 1.7 here as well, but note the negative exponent. It might help to rewrite choice B into y = 3( 1.7^(-2) )^x and note how b = (1.7)^(-2) = 0.346 approximately. This represents decay.

Choice C has b = 1/3 = 0.33 approximately which is also decay.

Finally, choice D has b = 2^(-1) = 1/(2^1) = 1/2 = 0.5 which is also decay.

Choices B through D have b values such that 0 < b < 1.

------------

Check out the graph below. It visually confirms the answers mentioned earlier. A growth function goes uphill as we move to the right, while a decay function moves downhill while moving to the right.

I used GeoGebra to make the graph.

Question Two please help

Answers

Answer:

The similarity statement is RTS and ACB. Other options are correct

Geometry, please answer question ASAP

Answers

9514 1404 393

Answer:

  D.  5.4

Step-by-step explanation:

The point U divides segment SP into the ratio 1 : 2. So, segment SU is 1/2 the length of segment UP.

  SU = UP/2 = 3.6/2 = 1.8

The length of PS is ...

  PS = PU +US = 3.6 +1.8

  PS = 5.4

__

Additional comment

If you were asked to find the value of x, you would discover x=1. That is not what you're asked here.

2 cm = 7.5 m in the drawing the mall is 8.6 cm tall how tall is the actual mall

Answers

Step-by-step explanation:

here's the answer to your question

If A=(prime number less than 10)list the element of set A and find n(A)​

Answers

A=(2 3 5 7)
N(A) =4


( sorry I need the points)

The product of two numbers is 10000.If one number is 16 times the other numbers ,find the two numbers.​

Answers

▓▓▓▓▓▓▓▓▓▓▓▓▓▓▓▓▓▓

➯Question:

The product of two numbers is 10000. If one number is 16 times the other numbers ,find the two numbers.

Answer:

Both numbers are 25 and 400.

Step By Step Explanation:

Given that:

The product of two numbers is 10000.

One number is 16 times the other number.

To Find:

Both numbers?

Solution:

Let us consider that one number be n, in question it is stated that other number is 16 times the first number. Therefore, other number is 16n.

According to the Question :

[tex]\sf↦ Product ~of ~numbers = 10000\\\\↦ n × 16n = 10000\\\\↦ 16n² = 10000\\\\↦ n² = \sf \dfrac{10000}{16}\\\\↦ \sf n = \sqrt{\dfrac{10000}{16}}\\\\↦ \sf n = \sqrt{\dfrac{100\:\times\:100}{4\:\times\:4}}\\\\↦ \sf n = {\cancel{\dfrac{100}{4}}}\\\\➦\pmb{\underline{\boxed{\sf{\pink{n = 25}}}}}[/tex]

Hence,

1st number = n = 25

2nd number = 16n = 16 × 25 = 400

∴ Hence, both numbers are 25 and 400.

▓▓▓▓▓▓▓▓▓▓▓▓▓▓▓▓▓▓

Answer:

(25, 400) or (-25, - 400)

Step-by-step explanation:

Let the numbers be x and y.

We have:

y = 16xxy = 10000

Substitute y and solve for x:

x(16x) = 1000016x² = 10000x² = 10000/16x² = 625x = √625x = ± 25

Then y is:

y = ± 25*16 = ± 400

The numbers are:

25 and 400 or -25 and - 400

Seventy-two books were to be divided up among 18 children. Which of the following
expressions is equivalent to this fraction?
a. 8/3 divided by 6/9
b. 8/3 times 2/3
c. 2/9 divided by 8/9
d. 2 times 8/2

Answers

Answer:

a

Step-by-step explanation:

cause if 72 book were divided by 18 kids they would hav all gotten 4 books each

so a is the answer cause it gives youthe answer 4

Answer: a. 8/3 divided by 6/9

Step-by-step explanation: I got this question on my test

Ashish is 175 cm tall his sister Annu is 8% shorter than him what is Annu's height

pls help​

Answers

Answer:

8% out of 175 = ( 8 ÷ 100 ) × 175 = 14

175 cm - 14 = 161 cm

...............

Fill in the table to explore the graph.
Miles Walked by Alonso
" Time (hours) 0.5 1.0 1.5 2.0 2.5 3.0 3.5 4.0
Miles Walked
6
V
16
12
Miles
8
4
X
1
4
3
Time (hours)
time

Answers

Answer:

Step-by-step explanation:

so you have divide  ok by my  and divide 5x1

[tex]\sqrt[3]{x+1} =2x+2[/tex]

Answers

the answer is down below

please help! will give brainliest if the answer is correct:) need the answers in order to move on in the lesson!

Answers

Answer:

Just use a calculator and wite the square root value in it

Step-by-step explanation:

Atempt this method with all options at last u will have ur answer

What is the domain of the function shown below?
[tex]y = log_{3}(x + 2) [/tex]
A. All real numbers greater than -2
B. All real numbers less than 2
C. All real numbers
D. All real numbers greater than 0​

Answers

Answer:

A

Step-by-step explanation:

A logarithmic function, log(x) has a domain x>0. For this logarithmic equation x+2>0, x>-2.

Zoe walks at a speed of 10 miles/h and jogs at a speed of 20 miles/h. She goes to the park to walk 2 miles on a Monday. How long will she take to walk 2 miles?

Answers

Answer:

12 minutes

Step-by-step explanation:

Given

[tex]s_1 = 10mi/h[/tex] --- walk speed

[tex]s_2 = 20mi/h[/tex] --- jog speed

[tex]d = 2\ miles[/tex] --- distance

Required

The time to walk the given distance

Time is calculated as:

[tex]Time = \frac{distance}{speed}[/tex]

In this case, the speed is the speed at which she walks.

So, we have:

[tex]Time = \frac{d}{s_1}[/tex]

Substitute known values

[tex]Time = \frac{2mi}{10mi/h}[/tex]

[tex]Time = 0.2hr[/tex]

Convert to minutes

[tex]Time = 0.2 * 60mins[/tex]

[tex]Time =12mins[/tex]

How do I solve this problem?

Answers

9514 1404 393

Answer:

  see attached

Step-by-step explanation:

Short answer: to solve this problem, follow directions.

a. Write the inequalities

Let c and p represent numbers of cakes and pies produced daily. Then the constraints are ...

  2c +3p ≤ 108 . . . . . . . . available hours of preparation time

  1c +0.5p ≤ 20 . . . . . . . available hours of decoration time

  p ≥ 0, c ≥ 0 . . . . . . . . . negative numbers of cakes or pies cannot be produced

__

b. Sketch the feasible region

It works well to let a graphing calculator do this.

The feasible region is the doubly-shaded area with vertices ...

  (0, 0), (0, 36), (3, 34), (20, 0)

__

c. Write the profit function

Profit is $25 per cake and $12 per pie, so is ...

  p = 25c +12p

__

d. Determine maximum profit

The attached table shows the profit for the various mixes of cakes and pies in the feasible region. The most profit is had by production of cakes only.

The maximum profit is $500 per day for production of 20 cakes.

_____

Additional comment

We have used x for cakes and y for pies in the attachment, because those are variables that the Desmos calculator prefers.

As is sometimes the case, the production point giving maximum profit leaves one of the resources (preparation time) only partially utilized.

Two thirds of the students in a class have pets. There are
36 students in the class. How many of the students have pets?

Answers

I am so sorry if this is wrong. But I hope it helps u.


Answer is - 24

I did 2/3x 36 =24
Answer:

24 of the students have pets

Step-by-step explanation:

Two thirds of the students in a class have pets =

2/3 of the students in a class have pets

If there’re 36 students in the class, how many of the students have pets?

Step 1. Convert 2/3 as a decimal

2/3 = 66.6

Step 2. Use part/whole = part/whole

66.6%/100% = ?/36

Let ? represent the amount of students who have pets in class

Step 3. Divide 66.6/100

0.666

Step 4. Then multiply it by 36

24
••••••••••••••••••••••••••••••••••••
doomdabomb: All brainliest
and thanks are appreciated
and would mean a lot to me,
thanks!

what is the ordered pair to (-4,2)

Answers

Answer:

(1,2) is ordered pair to (-4,2).

Step-by-step explanation:

(1,2)

hora khoi thaha vayena hai

Simplify : -2a(a-d)+2a(2a-3D) pls help.

Answers

-2a(a - d) + 2a(2a - 3d)

(-2a * a) + (-d * -2a) + (2a * 2a) + (2a * -3d)

-2a^2 + 2ad + 4a^2 - 6ad

(4a^2 - 2a^2) + (2ad - 6ad)

2a^2 - 4ad

---The above expression is simplified, though not fully. The below expression is fully simplified.

2a(a - 2d)

Hope this helps!

Help me pls

Find the missing segment in the image below

Answers

Answer:

The answer is 12

Step-by-step explanation:

You see its because.

Answer:

?=12

Step-by-step explanation:

Using the Thalès's theorem:

[tex]\dfrac{6}{6+4}=\dfrac{?}{20} \\\\?*10=6*20\ (cross\ products)\\\\?=\dfrac{6*20}{10} \ dividing\ by\ 10\\\\?=12[/tex]

10-7+12 dived (2^3-6) hope you can answer

Answers

Answer:

The answer is 7.5

The baseball team has a double-header on Saturday. The probability that they will win both games is 34%. The probability that they will win the first game is 75%, What is the probability that the team will win the second game given that they have already won the first game?
A. 45%

B. 74%

C. 90%

D. 26%

Answers

Answer:

A. 45%

Step-by-step explanation:

34/75=45 = 45%

Hi!! Can you please help with this question and explain how to solve it??
Find the height in centimeters of a square pyramid with a volume of 144 cm3 and a base edge length equal to the height. Give the approximate answer rounded to 2 decimal places.

Answers

9514 1404 393

Answer:

  7.56 cm

Step-by-step explanation:

The volume is given by the formula ...

  V = 1/3b²h . . . . where b is the base edge length, and h is the height.

Here we have V = 144 cm³, and b = h. Filling in these values and solving for h, we get ...

  144 cm³ = 1/3h³

  432 cm³ = h³

  h = ∛432 cm ≈ 7.55953 cm

The height of the pyramid is about 7.56 cm.

Answer:

[tex]7.56cm[/tex]

Step-by-step explanation:

Given that,

base length = Height

so,

[tex]volume = \frac{1}{3} {b}^{2} h \\ [/tex]

We can also write this as( according to this question)

[tex]v = \frac{1}{3} \times {b}^{2} \times b \\ v = \frac{ {b}^{3} }{3} [/tex]

let's solve

[tex]v = \frac{ {b}^{3} }{3} \\ 144 = \frac{ {b}^{3} }{3} \\ 144 \times 3 = {b}^{3} \\43 2 = {b}^{3} \\ \sqrt[3]{432} = b \\ b = 7.55[/tex]

[tex]base = height = 7.55cm \\ [/tex]

= 7.56cm

Other Questions
Find the length of AD in the figure.A. 26 unitsB. 34 unitsC. 122 unitsD. 130 units Is there a pattern of similarity and repetition evident in the development of these motifs? Or are there some differences and variations in the way they develop? Do these patterns of repetition and variation create formal parallelisms that structure the film? PLS HELP ME ON THIS QUESTION I WILL MARK YOU AS BRAINLIEST IF YOU KNOW THE ANSWER!!A ___________________ summarizes all values in a data set and lists how many times they occur.A. tally chartB. data valueC. data summaryD. frequency table In a 2-digit number, the tens digit is 5 less than the units digit. If you reverse the number, the result is 7 greater than double the original number. Find the original number. 3. Define the following terms A) Transparent B) Translucent c) Opaque the actual length of wall c is 4 m . to represent wall c noah draw a segment 16cm long what scale is he using explain your reasoning Sue was paid $384 for working 32 hours. How many hours will she have to work to earn $672? help me pleaseeeeee!!!!!! (4*10^4)+(6*10^2) Standard form is the value of x=3 find the value of 4x-2x+3x Duane Miller wants to know what price home he can afford. His annual gross income is $67,200. He has no other debt expenses and expects property taxes and insurance to cost $320 per month. He knows he can get a 8.50%, 15 year mortgage so his mortgage payment factor is 9.85. He expects to make a 25% down payment. What is Duane's affordable home purchase price?a. $107,929.b. $158,793.c. $138,207.d. $209,139.e. $179,665. Explain how the revival of classical texts contributed to the development of the Renaissance in Italy. What is the volume of the solid? Let =3.14 What does a scientist get instead of big money if a circumference of a circle is 22cm.find it diameter take pie 22/7. A command economy is best described as an economy that:A. gives businesses complete freedom to produce and sell what theywant.B. gives the government complete control over production andprices.C. gives both businesses and the government a role in economicdecisions.D. gives people jobs based on their family history and role in thecommunitySUBMIT How have nations responded to global environmental challenges? Which of the following sentences uses an interjection correctly? That tower of cans is about to fall. oh my goodness! That tower of cans is about to fall, oh my goodness! That tower of cans is about to fall Oh my goodness! That tower of cans is about to fall, Oh my goodness! We did all homework ____ couple of hours PLZ HELP, GIVING BRAINLIEST!!!Find the length of the intercepted arc. pi/6, r = 2.5